Calculate the sum of each expression. Arrange the expressions in the order of their value from largest to smallest.
92 + ( -5 )
2+2

Answers

Answer 1

Answer:

6

Step-by-step explanation:

Answer 2

Answer:

92 + ( -5 ) , 2+2

Step-by-step explanation:

1) 92 + ( -5 )

= 92 - 5

= 87

2) 2 + 2 = 4


Related Questions

(basic algebra) please help me with question:

Answers

Read the graph and you should see that at 4 years, the green bar (U Boxer) becomes more then the red bar (cost of new computer.)

If L (-5,4),M(2,2), N(0,-3), S(-7,-1), what is the length of diagonal LN

Answers

The length of the diagonal LN would be 8.60 units.

To find the distance between the two coordinates, We use distance formula ;

[tex]d = \sqrt{(x_2 - x_1)^2 + (y_2-y_1)^2}[/tex]
Lets put the values of L and N,
d= [tex]\sqrt{(0 - -5)^2 + (-3-4)^2}[/tex]

d =[tex]\sqrt{25+49}[/tex]

d= [tex]\sqrt{74}[/tex]

d= 8.60 units

Learn more about Distance formula on:

https://brainly.ph/question/24546230

#SPJ10

Which expression is equivalent to |8|+|-7|​

Answers

Answer:

8 + 7.

Step-by-step explanation: It is absolute value.

Please help i dont get this at all so please i will highly appreciate it .Thank youuu!!!

Answers

Answer:

2nd box, 4th box and  5th box

Step-by-step explanation:

A website sells a dress at £20 and shoes at £25. The website has an offer: Buy a dress and shoes together and get 1 3 off the price. There is also a shipping and handling charge of £4 added at the end, after any discount. If Ruth buys both a dress and shoes, how much will she actually pay?

Answers

£36

20+25=45          Take away 13 and it's 32.   32+4 (the shipping charge) is 36. So Ruth pays 36 pounds.

At a pizza restaurant it costs $4 to make a small pizza that sells for $12, and it costs $6 to make a large pizza that
sells for $15. In one week, the restaurant spent a total of $1100 to make pizzas and sold all of them for $2910. How
many small pizzas were sold?
small pizzas were sold?

Answers

simply just divide there total investment for pizza with the cost for one small pizza 1100/4=275

275 Small pizzas were sold

The sum of the series: -2+(-8) + (-32)++ (-2048) equals

I really neeed help!!! Please I’m begging you!!!!!!

Answers

Answer:

-2090

Step-by-step explanation:

1)  Remove parentheses.

-2 -8 -32 -2048

2) Simplify - 2 - 8 to -10.

-10 - 32 - 2048

3) Simplify -10-32 to -42.

-42 - 2048

4) Simplify.

-2090

image Which of the following statements is true for ∠a and ∠b in the diagram?

Answers

Answer:

You have not added a picture soi can not help answer this question.

Step-by-step explanation:

Add the picture by either taking a photo or screenshot of the question then upload it.

Solve for 'x' in the following equation, explaining each step.

3^ (-3x+7) = 3^ (7x-13)

Answers

Answer:

x = 2

Step-by-step explanation:

In order to have the two sides of the equation be equal, the exponents must equal.

-3x + 7 = 7x - 13

Add 3x to both sides.

7 = 10x - 13

Add 13 to both sides.

20 = 10x

Divide by 10.

2 = x

Check:

3^(-3•2+7)=3^(7•2-13)

3^1 = 3^1

3 = 3

Which equation best describes this line?

Answers

The answer is probably C

Find the range of f(x) = 3x + 2 for the domain = {1, 2, 3}.

Answers

Answer:

The domain of the expression is all real numbers except where the expression is undefined. In this case, there is no real number that makes the expression undefined.Interval Notation:..

(-∞,∞)

Set-Builder Notation:

{x|x∈ℝ}

The range is the set of all valid yy values. Use the graph to find the range…

Interval Notation:

(-∞,∞)

Set-Builder Notation:

{y|y∈R}

Determine the domain and range…

Domain: (−∞,∞),{x|x∈R}

Range:   (−∞,∞),{y|y∈R}

Answer:

{ 5, 8, 11 }

Step-by-step explanation:

the range is the corresponding values of f(x) for the values of x in the domain.

substitute the x- values from the domain into f(x)

f(1) = 3(1) + 2 = 3 + 2 = 5

f(2) = 3(2) + 2 = 6 + 2 = 8

f(3) = 3(3) + 2 = 9 + 2 = 11

range is { 5, 8, 11 }

While traveling to Europe, Phelan exchanged 250 US dollars for euros. He spent 150 euros on his trip. After returning to the United States he converts his money back to US dollars. How much of the original 250 US dollars does Phelan now have? Round to the nearest cent.

1 European euro = 1.3687 US dollars

Answers

The amount 44.70 US dollars

The graph represents the piecewise function:

Answers

Answer:

in the attachment

Step-by-step explanation:

Given: Piece-wise function graph.

We need to find the function.

In the graph, we can see a two-piece

For -3≤x≤0

Endpoints on graph (-3,-6) and (0,0)

It is the horizontal line. The equation of the horizontal line is y=k.

Equation: y=3

Now, we write as a piecewise function.

First blank = 2x

Second Blank = 3

hope this helps:)

please solve and tell me all that apply

Answers

Answer:

-x+4, 4+-x

Step-by-step explanation:

Obtain these answers by moving the X and 4 around and multiplying the equation by -1

HELP THIS IS URGENT I NEED IT SOON! PLEASE
If f(x) = x2 − 6x − 4 and g(x) = 5x + 3, what is (f + g)(−3)?
41
35
11
−35

Answers

Answer:

11

Step-by-step explanation:

(f + g)(x)

= f(x) + g(x)

= x² - 6x - 4 +5x + 3 ← collect like terms

= x² - x - 1

then

(f + g)(- 3) = (- 3)² - (- 3) - 1 = 9 + 3 - 1 = 11

Answer:

11

Step-by-step explanation:

F(–3)=(–3)²–6(–3)–4===> 9+18–4=23

g(–3)=5(–3)+3=‐15+3=–12

23–12=11

A company makes 140 bags. 39 of the bags have buttons but no zips. 21 of the bags have zips but no buttons. 23 of the bags have neither zips nor buttons. How many bags have zips on them?

Answers

Subtraction is a mathematical operation that reflects the removal of things from a collection. The number of bags that zips on them is 78.

What is subtraction?

Subtraction is a mathematical operation that reflects the removal of things from a collection. The negative symbol represents subtraction.

Given the number of bags that the company makes is 140. 39 of the bags have buttons but no zips. 21 of the bags have zips but no buttons. 23 of the bags have neither zips nor buttons.

Bags that have both zip and button

= Total number of bags - Number of bags with zips but no buttons - Number of bags with buttons but no zips - Number of bags with nor zips nor the buttons

= 140 - 21 - 39 - 23

= 57

Now, the number of bags that has zips will be the bags that only have zips and the number of bags that has zips and buttons. Therefore, the number of bags with zips is,

n = Bags that have both zip and button + Number of bags with zips but no buttons

   = 37 + 21

   = 78

Hence, the number of bags that zips on them is 78.

Learn more about Subtraction:

https://brainly.com/question/1927340

#SPJ1

Answer:

Bags with zip = (bag with button and zip) + (bag with no button but only zip)

i.e.,

Rotal bags – (bags with neither of two + bags with only buttons)

=> 140-(39+23)

=> 140-62

=> 78

So, 78 bags have zips on them.

several people are sitting evenly spaced around a circular table and the positions are labelled 1, 2 ,3 and so on .The people at positions 3 and 15 are directly opposite each other how many people are sitting at the table

Answers

12 will be opposite to 24. Then the number of the people are sitting at the table will be 24.

What is Algebra?

The analysis of mathematical representations is algebra, and the handling of those symbols is logic.

The several people are sitting evenly spaced around a circular table and the positions are labelled 1, 2, 3 and so on.

The people at positions 3 and 15 are directly opposite each other.

Then the number of the people are sitting at the table will be

1 is opposite to 13

2 is opposite to 14

3 is opposite to 15

Similarly, 12 will be opposite to 24.

Then the number of the people are sitting at the table will be 24.

More about the Algebra link is given below.

https://brainly.com/question/953809

#SPJ1

Approximate the area under the curve y = x^2 from x = 0 to x = 3 using a Right Endpoint approximation with 6 subdivisions.


Will mark brainliest

Answers

Subdivide the interval [0, 3] into 6 subintervals of equal length, ∆x = (3 - 0)/6 = 1/2. Then the partition is

[tex]\left[0,\dfrac12\right] \cup \left[\dfrac12,1\right] \cup \left[1,\dfrac32\right] \cup \cdots \cup \left[\dfrac52,3\right][/tex]

The right endpoint of the [tex]i[/tex]-th subinterval is

[tex]r_i = \dfrac12 + (i-1)\times\dfrac12 = \dfrac i2[/tex]

with [tex]i\in\{1,2,3,\ldots,6\}[/tex].

Then the area under the [tex]y=x^2[/tex] on the interval [0, 3] is approximately

[tex]\displaystyle \int_0^3 x^2 \, dx \approx \sum_{i=1}^6 (r_i)^2 \Delta x = \frac18 \sum_{i=1}^6 i^2[/tex]

Recall that

[tex]\displaystyle \sum_{n=1}^N n^2 = \frac{N(N+1)(2N+1)}6[/tex]

Then the approximate value of the area is

[tex]\displaystyle \int_0^3 x^2 \, dx \approx \frac{6\times7\times13}{48} = \boxed{\frac{91}8}[/tex]

The actual value of the area is 9, so this approximation is an overestimation, as is always the case when using a right endpoint sum for an increasing function.

m∠a=(48−x)°m∠b=(9x−38)°m∠c=90°

Answers

Answer:

Option C:

m∠a = 38°m∠b = 52°m∠c = 90°

Step-by-step explanation:

Hello!

The sum of all angles in a triangle is 180°.

Add up all the angles and solve for x by setting the equation to 180°.

Solve for x∠a + ∠b + ∠c = 180°(48 - x)° + (9x - 38)° + 90° = 180°10° + 8x + 90° = 180°8x = 180° - 90° - 10°8x = 80°x = 10°

Now that we solved for x, we can plug it back into each equation to solve for each angle.

Angle A48° - x48° - 10°38°Angle B9x - 38°9(10°) - 38°90° - 38°52°Angle C90°

The answer is Option C:

m∠a = 38°m∠b = 52°m∠c = 90°

someone help me with rest please asap!!

Answers

Answer:

3. Definition of Midpoint

4. VAT, Vertical Angles Theorem

5. ASA, Angle-Side-Angle Theorem

Please help answer this question asap 2/11

Answers

The correct answer is option A which is the value a₃₂ = 7.

What is the matrix?

A matrix is a specific arrangement of items, particularly numbers. A matrix is a row-and-column mathematical structure. The a_{ij} element in a matrix, such as M, refers to the i-th row and j-th column element.

In the given matrix the value of a₃₂ means that the element of the third row and the second column.

So the value of a₃₂ will be 7. which is present in the third row and the second column.

Therefore the correct answer is option A which is the value a₃₂ = 7.

More about the matrix link is given below.

https://brainly.com/question/9967572

#SPJ1

Please help!!!!! ASAP, If you can, i would love the steps on how to do it!! Thanks

Answers

Answer:

(5,18)  & (-5 , 38)

Step-by-step explanation:

Solutions of linear and quadratic system of equations:

  A linear equation represented by a line in a graph intersect the quadratic equation represented by a parabola zero, one or two times.

f(x) = x² - 2x + 3   -------------------(I)  

f(x) = -2x + 28   ---------------------(II)

x² - 2x + 3 = -2x + 28

x² - 2x + 2x + 3 - 28 = 0

x² - 25 = 0

x²         = 25

         x = √25 = √5*5

        x = ± 5

Plugin x = 5 in equation (II),

                    f(5) = -2*5 + 28

                          = -10 + 28

                          = 18

(5 , 18)

Plugin x = -5 in equation (II),

 f(-5) = -2*(-5) + 28

         = 10 + 28

         = 38

(-5, 38)

                         

Answer:

(5, 18) and (-5, 38)

Step-by-step explanation:

Given system of equations:

1) f(x) = x² - 2x + 3 ⇒ y = x² - 2x + 3

2) f(x) = -2x + 28 ⇒ y = -2x + 28

Solve using the Substitution Method:

Step 1: Substitute the the first equation into the second equation.

⇒ x² - 2x + 3 = -2x + 28

Step 2: Solve for x.

x² - 2x + 3 = -2x + 28 [ Add 2x to both sides. ]

x² - 2x + 2x + 3 = -2x + 2x + 28

⇒ x² + 3 = 28 [ Subtract 28 from both sides. ]

x² + 3 - 28 = 28 - 28

⇒ x² - 25 = 0 [ Factor using the following rule: a² - b² = (a - b)(a + b). ]  

(x - 5)(x + 5) = 0

⇒ x - 5 = 0, x + 5 = 0 [ Solve for the values of x. ]

x - 5 + 5 = 0 + 5, x + 5 - 5 = 0 - 5

x = 5, x = -5

Step 3: Solve for y.

Substitute the found x-values into one of the given equations.

y = x² - 2x + 3 ⇒ y = (5)² - 2(5) + 3 ⇒ y = 25 - 10 + 3 ⇒ y = 18

y = x² - 2x + 3 ⇒ y = (-5)² - 2(-5) + 3 ⇒ y = 25 + 10 + 3 ⇒ y = 38

The solutions to the system of equations are:

⟹ x₁ = 5, y₁ = 18 ⇒ (5, 18)

⟹ x₂ = -5, y₂ = 38 ⇒ (-5, 38)

Learn more about system of equations here:

brainly.com/question/26094713

brainly.com/question/27985265

8+6=?
what is the answer to the question

Answers

Answer: 14

Step-by-step explanation:

because 8 +4 is 14

14!, when you add 8 to 6 it gives you 14

An oatmeal container has a volume of 269 cubic inches. If the radius of the container is 3 cm, which statement below describes how to calculate the height?

Answers

The height of the volume of the container is 93.898cm.

We have given that,

An oatmeal container has a volume of 269 cubic inches.

Volume=269 cubic inches.

Radius=3cm

What is the formula for volume?

[tex]V=\pi r^2h[/tex]

[tex]269=\pi (3)^2h\\\269=9\pi h\\\h=\frac{269\pi }{9}\h=93.89871[/tex]

The height of the volume of the container is 93.898cm.

To learn more about the volume visit:

https://brainly.com/question/1972490

#SPJ1

Which linear function has the steepest slope?

Answers

Answer:

[tex]y=-8x+5[/tex]

Slope-intercept form:

[tex]\implies y=mx+b[/tex] [tex](\text{m=slope};\text{b=y-intercept})[/tex]

Using the slope-intercept form, let's identify the other slopes:

Option A:

[tex]y=-8x+5[/tex]  

[tex]\text{slope}=-8[/tex]

Option B:

[tex]y-9=-2(x+1)[/tex] (This equation is in point-slope form, the slope it too the left, making it -9)

[tex]\text{slope}=-9[/tex]

Option C:

[tex]y=7x-3[/tex]

[tex]\text{slope}=7[/tex]

Option D:

[tex]y+2=6(x+10)[/tex]

[tex]\text{slope}=2[/tex]  (Using point-slope terms, we can find the slope.)

⇒ A 'steep'est slope is closest to almost having a vertical slope or pitch, or  relatively high gradient, as a hill, an ascent, stairs, etc.

[tex]\text{This could be a line }[/tex] ⇒ ([tex]\text{positive or negative}[/tex])

Hence, the linear function with the steepest slope is Option A: [tex]y=-8x+5[/tex].

#SPJ2

Si un celular Cuesta 4500 y tiene un descuento de 76%cuamto vq a pagar por el teléfono

Answers

De acuerdo con las definiciones de precio a pagar, porcentaje de descuento y porcentaje remanente, el precio a pagar por el celular es $ 1080.

¿Cuánto cuesta un celular con descuento?

En esta situación una persona recibe un descuento por la compra de su celular, el precio a pagar es el producto de multiplicar el coste sin descuento y el porcentaje remanente en forma fraccionaria. El porcentaje remanente se calcula al restar el descuento del cien por ciento. A continuación, procedemos al cálculo requerido:

x = 4500 × (1 - 76/100)

x = 1080

De acuerdo con las definiciones de precio a pagar, porcentaje de descuento y porcentaje remanente, el precio a pagar por el celular es $ 1080.

Para aprender más sobre descuentos: https://brainly.com/question/24130820

#SPJ1

x varies directly as y. Find x when y=12 and k=6

Answers

72

x varies directly as y

hence x=k y

k=6

y=12

x=6×12

x=72

please help me please

Answers

Answer:

b I think.

Step-by-step explanation:

sorry if im wrong

What is equivalent? A B C OR D
please help

Answers

I think the answer should be C

Answer:

[tex]8^{\frac{x}{3} }[/tex]

Step-by-step explanation:

Hi 4 questions, about inequality’s and y intercepts and slope. Only 4 questions :)

Answers

The slope and y-intercept will be the negative 5/2 and 4.

The complete question is attached below.

What is the equation of line?

The equation of line is given as

y = mx + c

Where m is the slope and c is the y-intercept.

The equation is given below.

y > -(5/2)x + 4

The slope and y-intercept will be the negative 5/2 and 4.

The graph is also attached below.

More about the equation of line link is given below.

https://brainly.com/question/21511618

#SPJ1

Other Questions
Two neurons are shown below. In the drawing, label an axon, a dendrite, and a nucleus as well as a synapse A media personality argues that global temperatures are not rising, because every year an increase is reported, such as 0.09 degrees C. The difference from the previous year is less than the margin of error of about 0.13 degrees C, so that difference should be ignored. What is the best counterargument?The difference is only 0.04, but that is still a significant change.The margin of error is quite small compared to the change year over year, so it is a significant issue. The change of 0.09 is not a lot and thus should be ignored.The margin of error is larger than the increase, so we should consider it an issue of just extra information and thus can be ignored. What causes the chemical weathering of a rock containing iron to become soft and crumbly and reddish brown in color How effective is the author's response to the counterclaim that the job market for wind power is very limited?Effective; the author cites various fields of employment that are available in the wind power industry and supports the claim with a reputable source.Effective; the author presents a great deal of information regarding who benefits from wind power, including those seeking employment opportunities in the field.Ineffective; the author cites a source that is not reputable and facts that are unrelated to the counterclaim that the job market for wind power is limited.Ineffective; the author ignores the claim and provides unsupported evidence about the job market for wind power. one phone company charged 65% of its normal long-distance rate after 5 p.m. A day-rate long-distance call from Houston to Chicago costs 20 cents per minute. How much is an 11-minute call between these two cities after 5 p.m. ? Hey yall I need this answer quickly!!! 3/4 - 1/5 Which one of the following is caused by a direct blow to the body, overstretching, or excessivemuscle contraction?Complete strain Acute strainsChronic strainsAccurate strain Analyze In what way did both Vuk and Marconi contribute to Teslas outlooktoward humanity? (b) Make a Judgment Is Teslas disgust with humanityunderstandable? Why or why not? HELP FACTORISE ITS DUE IN TODAY PPLEASE Divide.74.48 7.6Enter your answer as a decimal in the box. 24. This week, Cristina eamed P594.65 from selling burgers for 35 days. How much he will earn in 5 days? A. P84.95 B. P94.95 C. P104.95 D. P114.95 25. You have four quizzes in your math subject in a quarter. You got 88.75 for the first quiz, 85.5, 90.5, and 87.25 in the second, third and fourth quizzes, respectively. What is your average in the four quizzes this quarter? A. 86.5 B. 88 C. 89.5 D. 90 26. Amberich put P580.00 into a savings account for one year. The rate of interest on the account was 6.5%. How much was the interest for one year in pesos and centavos? A. P67.70 B. P37.70 C. P57.70 D. P17.70 24. This week , Cristina eamed P594.65 from selling burgers for 35 days . How much he will earn in 5 days ? A. P84.95 B. P94.95 C. P104.95 D. P114.95 25. You have four quizzes in your math subject in a quarter . You got 88.75 for the first quiz , 85.5 , 90.5 , and 87.25 in the second , third and fourth quizzes , respectively . What is your average in the four quizzes this quarter ? A. 86.5 B. 88 C. 89.5 D. 90 26. Amberich put P580.00 into a savings account for one year . The rate of interest on the account was 6.5 % . How much was the interest for one year in pesos and centavos ? A. P67.70B. P37.70C. P57.70D. P17.70 4. What is the government of your Village/ town doing to keep its area clean? 4. What is the Village/town doing to keep its area clean ? Jack has a rectangular piece of land, the area of which is represented by a = 9.5%. His brother has a different rectangular piece of land, the area of which is represented by a2 = 14-). Let a represent the area in square meters and /represent the length in meters of the pieces of land. The two equations plotted on a graph meet at a point as shown in the image. If f(x)=2x +5(x-2), complete the following statement:The domain for f(x) is all real numbers ____ than or equal to 2.Free Response: lateral surface for a triangular prism l=8 w=4 h=6 given the formula p=k*2 w, find the value of p if k= 5 and w= - 3 the length of a cll is 2/3 mm. if the area of the cell is 1/12 square mm, what is the width of the cell As in Europe, growth of industrialization in the United States occurredO in large citiesO regionallyO by stateO nationallyQUICK IM DOING THE TEST RN Share your thoughts and habits when it comes to the nutrient called fat. Which option is an example of expository writing?